A lecture course consists of 595 students. The students are

This topic has expert replies
Moderator
Posts: 2207
Joined: Sun Oct 15, 2017 1:50 pm
Followed by:6 members

Timer

00:00

Your Answer

A

B

C

D

E

Global Stats

A lecture course consists of 595 students. The students are to be divided into discussion sections, each with an equal number of students. Which of the following cannot be the number of students in a discussion section?

A. 17
B. 35
C. 45
D. 85
E. 119

The OA is C.

Please, can anyone assist with above problem? Thanks in advance.

User avatar
GMAT Instructor
Posts: 15539
Joined: Tue May 25, 2010 12:04 pm
Location: New York, NY
Thanked: 13060 times
Followed by:1906 members
GMAT Score:790

by GMATGuruNY » Sat May 26, 2018 2:31 am
BTGmoderatorLU wrote:A lecture course consists of 595 students. The students are to be divided into discussion sections, each with an equal number of students. Which of the following cannot be the number of students in a discussion section?

A. 17
B. 35
C. 45
D. 85
E. 119
Number of discussion sections = (total number of students)/(number of students per section).
section.
Since the total number of students = 595, the number of students per section must be a FACTOR OF 595.

The question stem asks for a value that CANNOT be the number of students per classroom.
For an integer to be a multiple of 3, its digits must sum to a multiple of 3.
Since the sum of the digits of 595 is not a multiple of 3, 595 is not divisible by 3.
Implication:
Any answer choice that is a multiple of 3 cannot be a factor of 595 and thus cannot serve as the number of students per section.
Since 45 is a multiple of 3, it cannot serve as the number of students per section.

The correct answer is C. $$$$
Private tutor exclusively for the GMAT and GRE, with over 20 years of experience.
Followed here and elsewhere by over 1900 test-takers.
I have worked with students based in the US, Australia, Taiwan, China, Tajikistan, Kuwait, Saudi Arabia -- a long list of countries.
My students have been admitted to HBS, CBS, Tuck, Yale, Stern, Fuqua -- a long list of top programs.

As a tutor, I don't simply teach you how I would approach problems.
I unlock the best way for YOU to solve problems.

For more information, please email me (Mitch Hunt) at [email protected].
Student Review #1
Student Review #2
Student Review #3

GMAT/MBA Expert

User avatar
Elite Legendary Member
Posts: 10392
Joined: Sun Jun 23, 2013 6:38 pm
Location: Palo Alto, CA
Thanked: 2867 times
Followed by:511 members
GMAT Score:800

by [email protected] » Sat May 26, 2018 10:51 am
Hi All,

We're told that a lecture course consists of 595 students and the students are to be divided into discussion sections, each with an EQUAL number of students. We're asked which of the following CANNOT be the number of students in a discussion section. This question can be solved in a couple of different ways, including with Prime Factorization.

One way to determine all of the numbers that divide evenly into 595 is to prime-factor the total and list out the possible products from the factors:

595 =
(5)(119) =
(5)(7)(17)

We can now determine the list of factors of 595:
1, 5, 7, 17
(5)(7) = 35
(5)(17) = 85
(7)(17) = 119
595

You'll notice 4 of the answers are in the list and one is NOT...

Final Answer: C

GMAT assassins aren't born, they're made,
Rich
Contact Rich at [email protected]
Image

GMAT/MBA Expert

User avatar
GMAT Instructor
Posts: 1462
Joined: Thu Apr 09, 2015 9:34 am
Location: New York, NY
Thanked: 39 times
Followed by:22 members

by Jeff@TargetTestPrep » Wed May 30, 2018 4:23 pm
BTGmoderatorLU wrote:A lecture course consists of 595 students. The students are to be divided into discussion sections, each with an equal number of students. Which of the following cannot be the number of students in a discussion section?

A. 17
B. 35
C. 45
D. 85
E. 119
Any number whose digits sum to a number divisible by 3 is itself divisible by 3. For example, 3,912 is divisible by 3 because the sum of its digits is 3 + 9 + 1 + 2 = 15, which is divisible by 3.

Since 5 + 9 + 5 = 19, we see that 595 is not a multiple of 3. Since 45 is a multiple of 3, we cannot have 45 students in a discussion section.

Alternate solution:

Since 595 = 5 x 119 = 5 x 7 x 17, we see that 17 and 119 obviously can be the number of of students in discussion section and so can 35 (which is 5 x 7) and 85 (which is 5 x 17). So the only one it can't be is 45.

Answer: C

Jeffrey Miller
Head of GMAT Instruction
[email protected]

Image

See why Target Test Prep is rated 5 out of 5 stars on BEAT the GMAT. Read our reviews